1
$\begingroup$

Let $a_n>0$ and $b_n>0$ be two strictly declining sequences such that the series $$\sum_{n=1}^\infty \frac{a_n}{b_n}$$ is convergent. For $\sigma>0$ define $$f^N(\sigma) = \sum_{n=1}^N \frac{a_n}{b_n + \sigma/N}$$ Is it generally true that $\lim_{N \to \infty} f^N(\sigma)$ is independent of $\sigma$ or are there counterexamples?

Remarks:

  1. The answer is trivially true if $\sum \frac{a_n}{b_n^2}$ is convergent as well. In this case $$\left|\frac{d}{d\sigma} f^N(\sigma)\right| = \frac{1}{N}\sum_{n=1}^{N} \frac{a_n}{(b_n+\sigma/N)^2} \leq \frac{1}{N}\sum_{n=1}^N \frac{a_n}{b_n^2} \to 0$$
  2. More interesting is the case of divergent $\sum \frac{a_n}{b_n^2}$, e.g. $a_n = c^{-2n}$ and $b_n = c^{-n}$, or $a_n = 1/n^4$ and $b_n = 1/n^2$. In both these cases $$ \frac{d}{d\sigma} \left.f^N(\sigma)\right|_{\sigma=0} \to 1, $$ but from playing around with Maple and Mathematica I have the suspicion that $\frac{d}{d{\sigma}}f^N(\sigma)$ converges to $0$ for every $\sigma>0$, i.e. $f^N(\sigma)$ becomes non-differentiable in the limit. If that is true it would still allow for the possibility of $f^N(\sigma)$ converging pointwise to a constant.
  3. Eventually I am interested in the case $a_n = n^2I_n(K)^2$ and $b_n=I_n(K)$, where $I_n(K)$ is the modified Bessel function of the first kind.
  4. It might be related to the Stolz-Cesaro theorem, but I can't figure out how.

Any help or pointer to relevant literature is very much appreciated!

$\endgroup$

1 Answer 1

1
$\begingroup$

Since $\sum_ {n=1}^\infty \frac{a_n}{b_n } < \infty$ and $0 \le \frac{a_n}{b_n + \sigma/N}\le \frac{a_n}{b_n} $, we have that $\sum_{n=1}^N \frac{a_n}{b_n + \sigma/N} \to \sum_ {n=1}^\infty \frac{a_n}{b_n }$ as $N\to\infty$, just by dominated convergence.

$\endgroup$
2
  • $\begingroup$ Thanks! If I understand you correctly the dominated "function" in this case is defined by $g_{N,n} = a_n/(b_n+\sigma/N)$ for $n \leq N$ and $g_{N,n} = 0$ for $n > N$. Then $\sum_{n=1}^\infty g_{N,n} = f^N(\sigma)$ but also $g_{N,n} \to a_n/b_n$ pointwise. Since $g_{N,n}$ is dominated by $a_n/b_n$ the convergence follows by dominated convergence wrt to the counting measure. Very helpful! $\endgroup$
    – ingkanit
    Jan 11, 2013 at 18:21
  • $\begingroup$ Exact, it's the dominated convergence wrt to the counting measure (that one can state and prove directly, of course, without measure theory) $\endgroup$ Jan 11, 2013 at 18:26

Your Answer

By clicking “Post Your Answer”, you agree to our terms of service and acknowledge you have read our privacy policy.

Not the answer you're looking for? Browse other questions tagged or ask your own question.